Question regarding 1-cycles in the Collatz conjecture

collatz conjectureelementary-number-theoryproblem solving

The question is whether there is a simple explanation for the following observation, or whether the observation is an incorrect inference from limited cases. The context assumes 1-cycle loops in the Collatz conjecture (I know that Steiner proved that there are no 1-cycles; I am thinking about an old question from a distinguished reader regarding any approaches that do not require sophisticated relations among transcendental numbers or advanced theorems).

Assumptions:

(1) A loop of n elements, {$X_1, X_2,…X_n$} can be characterized by the equation $$\frac{3^n-2^n}{2^m-3^n}=X_1$$

(2) m is not free but is limited to integers that satisfy $nLog_2{3}<m<nLog_2{3}+1$.

(3) Every $X_i$ in the loop is associated with a number $b_i$ where $$\frac {b_i}{2^m – 3^n} = X_i$$

(4) For a 1-cycle, $b_1$ = $3^n – 2^n$

(5) For a 1-cycle, the $b_i$ are related by $$b_{i+1} = \frac {3}{2} * b_i + \frac {2^m – 3^n}{2}$$

Inferences:

(6) From (3) every $b_i$ must be divisible by $2^m – 3^n$.

(7) If two numbers a and b are divisible by d, then a-b is divisible by d.
Since the differences are divisible by d then the differences of the differences must be divisible by d, etc.

Observation:

(8) If we compute the appropriate values $b_i$ for a given n (and consequently by assumption (2), a pair {m,n}), then take the differences between consecutive $b_i$, then the differences of the differences, etc., we can construct tables such as the following:

For n=5 and m=8:

$b_i$ 1st diff 2nd diff 3rd diff 4th diff
211
323 112
491 168 56
743 252 84 28
1121 378 126 42 14

Following the same procedure for n=8, m=13 and we get:

$b_i$ 1st diff 2nd diff 3rd diff 4th diff 5th diff 6th diff 7th diff
6305
10273 3968
16225 5952 1984
25153 8928 2976 992
38545 13392 4464 1488 496
58633 20088 6696 2232 744 248
88765 30132 10044 3348 1116 372 124
133963 45198 15066 5022 1674 558 186 62

For n = 9 and m = 15 the last element is 126.

(9) The last element in the table is equal to $2^{m-n+1} – 2$

(10) The diagonal starting from the last element of the table and working backward proceeds in multiples of 2.

(11) The last row, starting from the last value, proceeds backward in multiples of 3.

(12) Stating from the last element we can fill out the table up to the first column. We can do this, for the ith column, 1 < i < n, by either by adding terms from the $i+1$ column to the diagonal elements that vary by factors of 2, or by subtracting terms from the $i+1$ column from the last row of the table, which vary by factors of 3. If we know the first element of the first column, we can fill out the rest by adding the terms in the second column.

(13) For the pair n=7, m=12 the difference is 5, the same for the pair n=8, m=13. The last element in the tables for each is 62.

(14) It would seem that any function in which the elements of the first column are related to each other in an orderly way will produce an orderly table; for example, if we start with the Fibonacci sequence, even if we start somewhere in the middle, the table will reproduce the earlier terms in the sequence:

21
34 13
55 21 8
89 34 13 5
144 55 21 8 3

Questions:

(14) Is it true that the last term in the 1-cycle related tables should be $2^{m-n+1} – 2$ for all permissible {n,m} pairs? By permissible I mean those which satisfy (2).

(15) If the answer to (14) is true, is there a straight-forward explanation for this?

(15) By (6) and (7) it would seem that every entry a given table would have to be divisible by $2^m – 3^n$ in order for there to be a 1-cycle. (Obviously 211 in the table for n=5 is a prime number, so there are no 5 element 1-cycles.) Is this true?

(16) Since all $b_i$ are odd numbers, and $2^{m-n+1} – 2$ is an even number, does this imply that $2^m – 3^n$ should also divide $2^{m-n} -1$, assuming that that the other assumptions regarding 1-cycles are satisfied?

(17) And I guess I should throw in, are any of the assumptions wrong as applied to 1-cycles in the Collatz conjecture?

(18) Any suggestions as to how to go about searching for answers to these on the internet?

ADDENDUM:

Thank you to Collag3n, Eric Shumard and Gottfried Helms. You are all very helpful. I didn't quite follow all of the math, but I did notice this in Mr. Shumard's response:

"from which $(2^{Pn}−3^n)|(2^{Pn−n}−1)$ follows", and from Collag3n's response:

"so in the end $2^m-3^n\nmid2^{m-n}-1$"

(19) So is this the kind of contradiction that demonstrates that there can be no 1-cycles?

Final Observations

(20) The equation given in (1) completely characterizes a 1-cycle for a given n; that is, given n we can compute all of the $X_i$ in the loop. This is because there is a one-to-one relationship between m and n for 1-cycles. This is not necessarily the case for any k-cycles other than 1. However, equations for the first 2 $b_i$ do completely describe these cycles, because the last term of the equation for $b_2$ is $2^{m-1}$.

(21) In finding values in the difference table that are not divisible by $2^m-3^n$ we can do consecutive divisions by 2 until we reach an odd number, rather than taking the difference of even numbers. The smallest difference between any two numbers in the difference table will then be an odd number which will have to be divisible by $2^m-3^n.$ This is true for any k-cycle or combination of k-cycles.

(22) For a 2-cycle, if it is possible to show that every difference table produces a number that is not divisible by $2^m – 3^n$, considering only those $b_i$ up to the point where $X_i > X_{i+1}$, i.e., the "break in the ascending sequence of the cycle, then there can be no 3-cycles, nor any other k-cycles, since the element of the difference table that precludes a 2-cycle will also be present in the difference table for higher order cycles. One may get away with including $b_{i+2}$ in computing the difference table.

Best Answer

Yes, you should have $2^m-3^n|2^{m-n}-1$ since $X_1+1=\frac{2^m-3^n+3^n-2^n}{2^m-3^n}=\frac{2^m-2^n}{2^m-3^n}$ and the denominator is odd so you can ignore the power of 2 in the numerator for the divisibility.

Notice that since $m<2n$ you can see that $2^{m-n}-1<2^n$, but we know (with Rhin's bound and for $n>5$) that $2^m-3^n>2^n$ so in the end $2^m-3^n\nmid2^{m-n}-1$

Also, I didn't try it myself, but it made me think of this Mathlologer video. Perhaps something you would want to explore: https://youtu.be/4AuV93LOPcE?t=2042